1968 AHSME Problems/Problem 16

Revision as of 03:13, 29 September 2014 by Timneh (talk | contribs) (Created page with "== Problem == If <math>x</math> is such that <math>\frac{1}{x}<2</math> and <math>\frac{1}{x}>-3</math>, then: <math>\text{(A) } -\frac{1}{3}<x<\frac{1}{2}\quad \text{(B) } -\f...")
(diff) ← Older revision | Latest revision (diff) | Newer revision → (diff)

Problem

If $x$ is such that $\frac{1}{x}<2$ and $\frac{1}{x}>-3$, then:

$\text{(A) } -\frac{1}{3}<x<\frac{1}{2}\quad \text{(B) } -\frac{1}{2}<x<3\quad \text{(C) } x>\frac{1}{2}\quad\\ \text{(D) } x>\frac{1}{2} \text{ or} -\frac{1}{3}<x<0\quad \text{(E) } x>\frac{1}{2} \text{ or } x<-\frac{1}{3}$

Solution

$\fbox{}$

See also

1968 AHSME (ProblemsAnswer KeyResources)
Preceded by
Problem 15
Followed by
Problem 17
1 2 3 4 5 6 7 8 9 10 11 12 13 14 15 16 17 18 19 20 21 22 23 24 25 26 27 28 29 30
All AHSME Problems and Solutions

The problems on this page are copyrighted by the Mathematical Association of America's American Mathematics Competitions. AMC logo.png